You are on page 1of 6

1

CN2116 (2110) Quiz One (Set One)


March 13 2010


All reactions are carried out at constant temperature and pressure unless indicated
otherwise.

1. Identify ALL the correct statements in the list of statements below. Please note that
each incorrect response will nullify a correct response.

(a) For first order reactions, the time taken to reduce the reactant concentration to 1/e
of the initial value is independent of the choice of the initial value.
(b) The (-1/r
A
) vs c
A
plot obtained from a laboratory CSTR can always be used for
the design of industrial PFRs so long as the feed concentration for the latter falls
within the range of concentrations studied in the laboratory.
(c) PFR is the reactor of choice for harvesting the intermediate product R in the first
order irreversible series reactions ARS. Hence CSTR should be used if the
desirable product is S.
(d) Recycle does not matter to CSTRs since the latter are already perfectly mixed.
This is true for CSTRs in series too.
(e) The rate law for the reversible reaction A B C D + + is
1 2 A B C D
r k c c k c c =

Solution

(a) TRUE. This value is 1/k and equal to the Damkholer number of 1
st
order
reactions
(b) TRUE. Since this is the plot of the rate law
(c) FALSE. S is the end product and hence the reactor of choice will depend on
whatever type that leads to a smaller overall volume. Since the reactions are all 1
st

order. PFR should still be used.
(d) FALSE: Series-connected CSTRs have acquired some PFR characteristics
(e) FALSE: Rate law cannot be predicted from reaction stoichiometry.

2. In a contracting reaction system mAR (where m>1) carried out at constant T,P, the
decrease in the concentration of A is partly compensated by the reduction in the
reaction volume. What is the maximum concentration of A possible in a contracting
reaction system?

(a) Greater than
0 A
c
(b) Equal to
0 A
c
(c) Smaller than
0 A
c
(d) None of the above

Solution (b)

2
The most negative value of
A
c is -1 (at
0
1, 1
A
y o = = ) hence
0 0
0
(1 ) (1 )
(1 ) (1 )
A A A A
A A
A A A
c x c x
c c
x x c

= = =
+


3. Consider the following reactions in an aqueous solution, which of the following
concentration relationships is correct?

2
0.5 2
A R S
T U
o
| o

+ + where , , and o | o are molar extents of reactions


( ) / 2 constant
( ) / 2 constant
( ) / 2 2 / 2 constant
( ) 2 / 2 2 constant
A R S
A R T
A R S T U
A R S T U
a c c c
b c c c
c c c c c c
d c c c c c
+ + =
+ + =
+ + + + =
+ + + + =


Solution (c)

0
0
2
0.5
2
/ 2 2 / 2
A A
R
S
T
U
A R S T U A
F F
F
F
F
F
F F F F F F
o |
o o

|
o
=
=
=
=
=
+ + + + =


4. Two CSTRs of sizes 1L and 2L are connected in series with the 2L reactor leading
the 1L reactor. The reaction of interest is an aqueous-phase reaction with negative
order kinetics. If it is desired to improve the efficiency of the reactor operation (i.e.
increasing the product concentration, product throughput, or both), which of the
following measures will you endorse?

(a) Convert the two CSTR into batch reactors
(b) Reverse the reactor sequence
(c) Connect the reactors in parallel
(d) Decease the operating temperature

Solution (c)

For negative order reactions, CSTRs are favored over PFRs. The reactors must be
arranged in such a way to quench the concentration as quickly as possible. Option (c)
corresponds to a 3L reactor which is more effective than the 2 reactors connected in
series in any sequence (see lecture notes)
3

5. What are the conditions needed to express the 2
nd
order kinetics
2
A
r kc = as
2 A
p A
dp
k p
dt
= in a batch reactor?

(a) unconditionally; (b) constant T, (c) constant V, (d) constant T and V

Solution (d)

2
2
1
,
1
A
A A
A A
A A
A A
dN
r kc
V dt
p V p
N c
RT RT
p V p d
k
V dt RT RT
= =
= =
| | | |
=
| |
\ . \ .

This will reduce to the form of
2 A
p A
dp
k p
dt
= only if T and V are kept constant

6. Determine the reaction order of a liquid phase reaction from the following
concentration-time curve
























4


(a) Second order, (b) first order; (c) zero order, (d) none of the above

Solution (a)

Take the times needed successively to halve the concentration from 1 to 0.5, and from
0.5 to 0.25 and use the following formula

'
1 1/ 2
1/ 2
1.5 0.5
2 , 2
0.5
n
t
n
t

= = =

7. Indicate which of the following contacting schemes would maximize the yield of R
per mole of A converted in the following competing reactions:

1 1
2
2 2 1 2
A B
A
A B R r k c c
A S r k c E E
+ =
= <


(a) Isothermal operation at lowest possible temperature in a plug flow reactor
(b) Isothermal operation at highest possible temperature in a mixed flow reactor
(c) Rising temperature profile, B in plug flow, A in mixed flow
(d) Decreasing temperature profile, B in plug flow, A in mixed flow

Solution (c)

We want c
B
to be high, c
A
to be low, and low temperature when reaction rate is high
(at reactor entrance), and high temperature when reaction rate is low (at reactor exit)
5

8 Which of the following reactor arrangements should be used to handle the reaction
with the following rate curve? c
A0
=10, c
A
= 1

























(a) A PFR from c
A0
to c
A
=4 followed by a CSTR from c
A
=4 to c
A
= 1
(b) A CSTR from c
A0
to c
A
=4 followed by a PFR from c
A
=4 to c
A
= 1
(c) A PFR from c
A0
to c
A
=1
(d) A recycle PFR from c
A0
to c
A
=4 with R=1

Solution (b)

9. Three unequal sized CSTRs (1L, 2L, and 3L respectively) are connected in parallel to
handle the conversion of a fresh feed to 90% completion. If the 2L reactor is all of a
sudden broken down, what will be the reduction in the product throughput if the
product quality is to be maintained at the same level as that before the breakdown?

(a) Loss of 10%
(b) Loss of 33.3%
(c) Loss of 50%
(d) Loss of 66.6%

Solution (b)
6
The initial reactor arrangement amounts to a 6l reactor, after the break down the size
is reduced to 4l. Hence the throughput loss is 1/3

10. At present 91% conversion of A is converted into product by a first order kinetics in a
single mixed flow reactor. We plan to add in series two more reactors of the same
size as the one in use. What will be the new treatment rate if the conversion is kept
unchanged?

(a) approximately 5 times; (b) approximately 8 times; (c) approximately 10 times; (d)
approximately 15 times

Solution (b)

From n=1 design chart
1 3
10, 3.7 k k t t = =

Since


1 3 '
0 0
'
o 0
3
,
v v
v 8.1v
V V
t t = =
=

You might also like